low levels of immune-system activity

This topic has expert replies
User avatar
Senior | Next Rank: 100 Posts
Posts: 79
Joined: Wed May 23, 2012 7:23 pm
Thanked: 10 times

low levels of immune-system activity

by umeshpatil » Sun Sep 23, 2012 10:14 am
A researcher discovered that people who have low levels of immune-system activity tend to score much lower on tests of mental health than do people with normal or high immune-system activity. The researcher concluded from this experiment that the immune system protects against mental illness as well as against physical diseases.

The researcher's conclusion depends on which of the following assumptions?
a. High immune-system activity protects against mental illness better than normal immune-system activity does.
b. Mental illness is similar to physical disease in its effects on body systems.
c. People with high immune-system activity cannot develop mental illness.
d. Psychological treatment of mental illness is not as effective as is medical treatment.

I will provide OA as soon as I get justification for the right answer and why others are wrong.

Junior | Next Rank: 30 Posts
Posts: 28
Joined: Tue Aug 21, 2012 6:10 pm
Thanked: 2 times

by SmartAssJun » Sun Sep 23, 2012 4:46 pm
umeshpatil wrote:A researcher discovered that people who have low levels of immune-system activity tend to score much lower on tests of mental health than do people with normal or high immune-system activity. The researcher concluded from this experiment that the immune system protects against mental illness as well as against physical diseases.

The researcher's conclusion depends on which of the following assumptions?
a. High immune-system activity protects against mental illness better than normal immune-system activity does.
b. Mental illness is similar to physical disease in its effects on body systems.
c. People with high immune-system activity cannot develop mental illness.
d. Psychological treatment of mental illness is not as effective as is medical treatment.

I will provide OA as soon as I get justification for the right answer and why others are wrong.
Are you sure there's only four answer choices to this question?
None of the four choices seem relevent to me.
But I'll have to choose B.

Junior | Next Rank: 30 Posts
Posts: 14
Joined: Wed Jan 12, 2011 9:45 am
Followed by:2 members

by ananthrajavadhuta » Sun Sep 23, 2012 10:19 pm
IMO : C

If option C is negated, argument falls apart.

User avatar
Senior | Next Rank: 100 Posts
Posts: 79
Joined: Wed May 23, 2012 7:23 pm
Thanked: 10 times

by umeshpatil » Tue Sep 25, 2012 7:21 pm
OA:C. Correct.

@ananthrajavadhuta, Does it mean, we need to use only negation test in such confusing questions ?

Newbie | Next Rank: 10 Posts
Posts: 8
Joined: Fri Sep 28, 2012 6:27 pm
Location: Tampa, Fl

by EricJA » Fri Sep 28, 2012 8:17 pm
None of the answers seems right to me. C is not an assumption the researcher can make based on the evidence, and negating it does not destroy the argument's conclusion that the normal or above immune system offers protection against mental illness. The argument says protection, not perfect protection. If c is the answer, shame on the folks who wrote the question. A condom offers protection, but wearing one does not guarantee you won't become a parent...so protection does not mean zero possibility of illness.

A is not warranted because only low immune system activity correlates with poorer mental health. Above average and average immune activity don't and there is no suggestion the above average immune system activity is any better than average immune system activity as far as better mental health.

B is so broad, but possibly better than A because at least the immune system working at average or above average levels produces the same effect for both physical and mental illnesses -- some degree of protection. The essence of the argument is that the same system works to prevent mental and physical illnesses so b seems true -- both kinds of illness activate some protection.

D. You can't get to D without some data on how effective non-medical treatments are compared to medical ones.

It's not a very well constructed question. :-)

User avatar
Senior | Next Rank: 100 Posts
Posts: 72
Joined: Sat May 19, 2012 2:27 pm
Thanked: 2 times
Followed by:1 members

by Sapana » Wed Jan 09, 2013 12:12 pm
One more option fort his question is missing which probably is the answer because none of the choice seem good to me as well.

User avatar
Master | Next Rank: 500 Posts
Posts: 194
Joined: Mon Oct 15, 2012 7:14 pm
Location: India
Thanked: 47 times
Followed by:6 members

by The Iceman » Thu Jan 10, 2013 6:15 am
Sapana wrote:One more option fort his question is missing which probably is the answer because none of the choice seem good to me as well.
Indeed!The right option is not even provided!

@Umesh...please ALWAYS post all 5 choices(especially the right one ;)) whenever you post a problem.

User avatar
GMAT Instructor
Posts: 2193
Joined: Mon Feb 22, 2010 6:30 pm
Location: Vermont and Boston, MA
Thanked: 1186 times
Followed by:512 members
GMAT Score:770

by David@VeritasPrep » Thu Jan 10, 2013 9:00 am
THE CORRECT VERSION OF THE QUESTION IS HERE

Sapana, Ice Man, you are correct the question given in the first posting is not the complete question at all! Which is surprising since this is from The Official Guide for GMAT Verbal Review, 2nd Edition (page 118, question #7).

Ignore the question as written above, Here is the correct question:

"A researcher discovered that people who have low levels of immune-system activity tend to score much lower on tests of mental health than do people with normal or high immune-system activity. The researcher concluded from this experiment that the immune system protects against mental illness as well as against physical disease.

The researcher's conclusion depends on which of the following assumptions?

(A) High immune-system activity protects against mental illness better than normal immune-system activity does.
(B) Mental illness is similar to physical disease in its effects on body systems.
(C) People with high immune-system activity cannot develop mental illness.
(D) Mental illness does not cause people's immune-system activity to decrease.
(E) Psychological treatment of mental illness is not as effective as is medical treatment."

The official answer is D.


There is a great discussion of the question on the Veritas Prep Blog. One of our veteran instructors, Antony Ritz, not only breaks down the causation assumption in this question, but also points you to several other official questions that you can practice causation on!
here is the link: https://www.veritasprep.com/blog/2012/10 ... -part-iii/

Let me know if you have any questions after reading the blog post.
Veritas Prep | GMAT Instructor

Veritas Prep Reviews
Save $100 off any live Veritas Prep GMAT Course

Junior | Next Rank: 30 Posts
Posts: 10
Joined: Tue Nov 05, 2013 6:31 am

by tusharkhatri123 » Thu Jan 15, 2015 1:20 am
Hi Experts,
Although I got that option D is incorrect due to reverse causation, I don't understand why option B is incorrect.
According to me int he first sentence of the premise, only mental health is discussed and not physical disease. So how can we include physical disease along with mental illness in the conclusion part? For this, according to me, we have to assume that mental illness and physical disease are similar. This is given in option B.

Please correct me wherever I am going wrong.

Thanks
Tushar

User avatar
GMAT Instructor
Posts: 15539
Joined: Tue May 25, 2010 12:04 pm
Location: New York, NY
Thanked: 13060 times
Followed by:1906 members
GMAT Score:790

by GMATGuruNY » Thu Jan 15, 2015 7:14 am
tusharkhatri123 wrote:Hi Experts,
Although I got that option D is incorrect due to reverse causation, I don't understand why option B is incorrect.
According to me int he first sentence of the premise, only mental health is discussed and not physical disease. So how can we include physical disease along with mental illness in the conclusion part? For this, according to me, we have to assume that mental illness and physical disease are similar. This is given in option B.

Please correct me wherever I am going wrong.

Thanks
Tushar
A conclusion is an assertion that might -- or might not -- be true.
By definition, the immune system is the body's defense against infectious organisms and other foreign invaders.
Since it is a generally accepted FACT that the immune system protects against physical disease, this assertion is not part of the conclusion.
What might or might not be true is the following assertion:
The immune system ALSO PROTECTS AGAINST MENTAL ILLNESS.
Since the conclusion is limited to the link between the immune system and mental illness, answer choice B is irrelevant.
Private tutor exclusively for the GMAT and GRE, with over 20 years of experience.
Followed here and elsewhere by over 1900 test-takers.
I have worked with students based in the US, Australia, Taiwan, China, Tajikistan, Kuwait, Saudi Arabia -- a long list of countries.
My students have been admitted to HBS, CBS, Tuck, Yale, Stern, Fuqua -- a long list of top programs.

As a tutor, I don't simply teach you how I would approach problems.
I unlock the best way for YOU to solve problems.

For more information, please email me (Mitch Hunt) at [email protected].
Student Review #1
Student Review #2
Student Review #3

Junior | Next Rank: 30 Posts
Posts: 10
Joined: Tue Nov 05, 2013 6:31 am

by tusharkhatri123 » Thu Jan 15, 2015 9:54 am
Hi Mitch Sir,
You said the following:
"Since it is a generally accepted FACT that the immune system protects against physical disease"
I have learnt that in every critical reasoning question, while solving we don't have to consider anything. So if you have considered the above sentence as a pre-assumed fact, then it's against the rule. Please clarify my doubt.

Thanks
Tushar

User avatar
GMAT Instructor
Posts: 15539
Joined: Tue May 25, 2010 12:04 pm
Location: New York, NY
Thanked: 13060 times
Followed by:1906 members
GMAT Score:790

by GMATGuruNY » Mon Jan 19, 2015 4:55 am
tusharkhatri123 wrote:Hi Mitch Sir,
You said the following:
"Since it is a generally accepted FACT that the immune system protects against physical disease"
I have learnt that in every critical reasoning question, while solving we don't have to consider anything. So if you have considered the above sentence as a pre-assumed fact, then it's against the rule. Please clarify my doubt.

Thanks
Tushar
When we evaluate a CR, we may not consider specialized knowledge -- knowledge known only by a special few.
But we are EXPECTED to have common sense and to consider COMMON knowledge -- knowledge known by virtually everyone.
It is common knowledge that THE IMMUNE SYSTEM PROTECTS AGAINST PHYSICAL DISEASE.
Private tutor exclusively for the GMAT and GRE, with over 20 years of experience.
Followed here and elsewhere by over 1900 test-takers.
I have worked with students based in the US, Australia, Taiwan, China, Tajikistan, Kuwait, Saudi Arabia -- a long list of countries.
My students have been admitted to HBS, CBS, Tuck, Yale, Stern, Fuqua -- a long list of top programs.

As a tutor, I don't simply teach you how I would approach problems.
I unlock the best way for YOU to solve problems.

For more information, please email me (Mitch Hunt) at [email protected].
Student Review #1
Student Review #2
Student Review #3

User avatar
Legendary Member
Posts: 2131
Joined: Mon Feb 03, 2014 9:26 am
Location: https://martymurraycoaching.com/
Thanked: 955 times
Followed by:140 members
GMAT Score:800

by MartyMurray » Mon Jan 19, 2015 2:08 pm
Also, even if one did not understand that the immune system protects against physical disease, B is still irrelevant because of the following.

The conclusion is that the immune system protects against mental illness, and, in one possible way of reading it, physical illness. In other words, the conclusion is one of cause and effect, the cause being higher immune system activity and the effect being better mental health, and, according to your reading, physical health.

B does not discuss anything that would either underpin or call into question this cause and effect relationship. Instead it discusses the degree of similarity between the effects of mental disease and those of physical disease. So it is irrelevant.
Marty Murray
Perfect Scoring Tutor With Over a Decade of Experience
MartyMurrayCoaching.com
Contact me at [email protected] for a free consultation.